CPR 1 questions

अब Quizwiz के साथ अपने होमवर्क और परीक्षाओं को एस करें!

115 A 35-year-old man is admitted to the hospital with severe chest pain, dyspnea, tachycardia, cough, and fever. Radiologic examination reveals significant pericardial effusion. When pericardiocentesis is performed, the needle is inserted up from the infrasternal angle. The needle passes too deeply, piercing the visceral pericardium and entering the heart. Which of the following chambers would be the first to be penetrated by the needle? A. Right ventricle B. Left ventricle C. Right atrium D. Left atrium E. The left cardiac apex

115 A. Pericardiocentesis is usually performed through the infrasternal angle with the needle passing up through the diaphragm to the fibrous pericardium. The diaphragmatic surface of the heart is largely com- posed of the right ventricle and would therefore be entered if a needle is inserted too far. The other cham- bers of the heart would not lie in the direct path of the needle.

116 A 45-year-old man is admitted to the hospital with severe chest pain radiating to his left arm and left upper jaw. An emergency ECG reveals an acute myocardial infarction of the posterior left ventricular wall. Which of the following spinal cord segments would most likely receive the sensations of pain in this case? A. T1, T2, T3 B. T1, T2, T3, T4 C. T1, T2 D. T4, T5, T6 E. T5, T6, T7

116 B. The pain experienced by the patient travels with the sympathetic innervation of the heart, derived from spinal nerve levels T1 to T4. The pain fibers leave the heart and the cardiac plexuses via the car- diopulmonary nerves. Subsequently, the pain fibers pass through the sympathetic chain, enter the spinal nerve, and pass into the dorsal roots of the spinal nerves. The cell bodies of the pain fibers are located in the dorsal root ganglia of the spinal nerves from T1 to T4. The other levels indicated do not correspond to the typical pattern of innervation of this region

12 A 3-year-old male patient presents with a clinically significant atrial septal defect (ASD). The ASD usually results from incomplete closure of which of the following structures? A. Foramen ovale B. Ligamentum arteriosum C. Ductus arteriosus D. Sinus venarum E. Coronary sinus

12 A. An ASD is a communication between the right and left atria. In the formation of the partition between the two atria, the opening in the foramen secundum, also known as the foramen ovale, typically closes at birth. If it remains patent, an ASD will result. The rest of the structures are not associated with ASDs.

121 A 35-year-old woman is admitted to the hospital with a complaint of shortness of breath. During physical examination it is noted that there is wide splitting in her S2 heart sound. Which of the following valve(s) is/are responsible for production of the S2 heart sound? A. Mitral valve B. Pulmonary and aortic C. Aortic and mitral D. Tricuspid E. Tricuspid and aortic

121 B. The S2 heart sound refers to the second (dub) heart sound. This sound is produced by the closure of the aortic and pulmonary semilunar valves. The closure of mitral/bicuspid and tricuspid valves produce the first S1 (lub) heart sound.

122 A 35-year-old woman is admitted to the hospital with dyspnea. During physical examination her S1 heart sound is very loud. Which of the following valve(s) is/ are responsible for production of the S1 heart sound? A. Mitral valve B. Pulmonary and aortic C. Aortic and mitral D. Tricuspid E. Tricuspid and mitral

122 E. The closure of the mitral/bicuspid and tricuspid valves produces the first S1 (lub) heart sound. The S2 heart sound refers to the second (dub) heart sound. This latter sound is produced by the closure of the aortic and pulmonary semilunar valves.

125 A 62-year-old woman accountant is admitted to the emergency department with severe chest pains that radiate to her left arm. ECG reveals that the patient suffers from an acute myocardial infarction. Coronary angiography is performed and a stent is placed at the proximal portion of the anterior interventricular artery (left anterior descending). Because of the low ejection fraction of the right and left ventricles, a cardiac pacemaker is also placed in the heart. The function of which of the following structures is essentially replaced by the insertion of a pacemaker? A. AV node B. SA node C. Purkinje fibers D. Bundle of His E. Bundle of Kent

125 B. The SA node functions as the primary intrinsic pacemaker of the heart, setting the cardiac rhythm. An artificial pacemaker assists in producing a normal rhythm when the SA node is not functioning normally. The atrioventricular node receives the depolarization signals from the SA node. The signal is delayed within the atrioventricular node (providing the time for the atria to contract), then propagated from the atrioventricular node through the bundle of His and Purkinje fibers.

139 A 27-year-old male billiards player received a small-caliber bullet wound to the chest in the region of the third intercostal space, several centimeters to the left of the sternum. The patient is admitted to the emergency department and a preliminary notation of "Beck's triad" is entered on the patient's chart. Which of the following are features of this triad? A. There was injury to the left pulmonary artery, left primary bronchus, and esophagus B. The patient has bleeding into the pleural cavity, a collapsed lung, and mediastinal shift to the right side of the thorax C. The patient has a small, quiet heart; decreased pulse pressure; and increased central venous pressure D. The young man is suffering from marked diastolic emptying, dyspnea, and dilation of the aortic arch E. The left lung has collapsed, there is paradoxical respiration, and there is a mediastinal shift of the heart and trachea to the left

139 C. The patient is suffering from cardiac tamponade, that is, filling of the pericardial cavity with fluid. The classic signs of this tamponade are referred to as "Beck's triad." This trio, by definition, includes a small heart from compression of the heart by the fluid-filled pericardial sac, and a quiet heart because the tamponade muffles the cardiac sounds; decreased pulse pressure resulting from the reduced difference between systolic and diastolic pressure because the tamponade restricts the ability of the heart to fill in diastole; and increased central venous pressure because venous blood cannot enter the compressed heart.

141 Two days after the patient's breathing had become assisted by mechanical ventilation, a patient with Guillain- Barr. syndrome began experiencing severe cardiac arrhythmia, with perilously slow cardiac contractions, resulting in reduced cardiac output. This most likely resulted from interruption of the contractile stimulus carried by which of the following? A. Left vagus nerve B. Right phrenic nerve C. Preganglionic sympathetic fibers in upper thoracic spinal nerves D. Cardiac pain fibers carried by upper thoracic spinal nerves E. Ventral horn neurons of spinal cord levels T1 to T4

141 C. The loss of myelin from the preganglionic (normally myelinated) sympathetic fibers in T1 to T4 results in interruption in their transmission of electri- cal stimulating impulses and, therefore, reduction of positive inotropic (force increasing) and chronotropic (rate increasing) stimulation of the heart. Reduction of function of the vagus nerves would not result in slowing cardiac activity; just the opposite would occur. Interruption of phrenic nerve activity has no effect on cardiac rate (as this nerve innervates the diaphragm), nor would the interruption of the thinly myelinated pain fibers from the heart. The ventral horn neurons do not innervate the heart, but rather skeletal muscle; therefore, they would not be directly affected by the disease process affecting the heart.

142 During transesophageal echocardiography (TEE), an ultrasound transducer is placed through the nose or mouth to lie directly behind the heart. The closer a structure is to the transducer, the better the ultrasound image that can be obtained. In TEE, which heart valve can be best visualized? A. Tricuspid B. Pulmonary C. Mitral D. Aortic E. Valve of the inferior vena cava

142 C. The mitral valve is best visualized by TEE because the transducer within the esophagus is directly posterior to the left atrium. The physical laws that apply to ultrasound imaging dictate that the closer the structure to the transducer, the better the ability to obtain a good image. This question asks which heart valve is most directly related to the posterior aspect of the left atrium, which is the mitral valve.

151 A 51-year-old woman visited her physician complaining of dyspnea. Examination revealed edema of the lower limbs and a systolic murmur was heard in the left second intercostal space. Which of the following valve abnormalities is she most likely suffering from? A. Regurgitation through aortic valve B. Regurgitation through pulmonary valve C. Stenosis of aortic valve D. Regurgitation through mitral valve E. Stenosis of pulmonary valve

151 E. Pulmonary stenosis of the pulmonary valve results in a systolic murmur that can be auscultated at the left second intercostal space. During systole, blood is forcibly expelled from the ventricles and result in turbulent flow against a narrowed valve. Aortic stenosis also results in a systolic murmur but is auscultated at the right second intercostal space. Regurgitation through the mitral valve results in a systolic murmur that is auscultated at the fourth left intercostal space in the midclavicular line. Pulmonary and aortic valve regurgitation result in diastolic murmurs.

154 A 65-year-old woman is admitted to the emergency department with severe chest pain. Laboratory examination reveals that the patient has suffered a myocardial infarction. Cardiac catheterization is performed and the portion of the heart immediately behind the sternum is found to be infarcted. Which artery is most likely occluded? A. Circumflex B. Anterior interventricular C. Posterior interventricular D. Left marginal E. Right coronary

154 E. The right coronary artery supplies the right ventricle, which lies immediately posterior to the sternum. The circumflex artery supplies the left atrium. The anterior interventricular artery supplies the left ventricle and the anterior two thirds of the interventricular septum. The posterior interventricular artery supplies the posterior third of the interventricular septum. The left marginal artery is a branch of the circumflex artery and supplies the posterior part of the left ventricle.

157 A 25-year-old man is admitted to the hospital with severe headache, cold feet and legs, and pain in his legs when he runs a short distance. During physical examination, femoral pulses are much weaker than radial pulses. Three-dimensional CT scan angiography reveals a coarctation of the aorta proximal to the left subclavian artery. The condition that creates these symptoms is a result of a failure of normal development of which structure? A. Fourth pharyngeal arch B. Third pharyngeal arch C. Left dorsal aorta D. Left fifth pharyngeal arch E. Sixth pharyngeal arch

157 A. During embryonic development the left dorsal aorta gives rise to the thoracic aorta. The aortic arch is formed by the aortic sac and the fourth pharyngeal artery. The third pharyngeal artery will give rise to the common carotid artery. The fifth pharyngeal arch artery will disappear bilaterally, and the sixth will form the ductus arteriosus on the left and part of the pulmonary trunk. In this case the region of the arch of the aorta between the subclavian artery and the left common carotid artery is formed by the fourth aortic arch.

160 A 60-year-old woman with a history of severe rheumatic heart disease is in the clinic for a routine follow-up. Her cardiac symptoms have been relatively stable since valve replacement surgery 2 years ago, but she has developed some new symptoms over the past few months. A chest radiograph reveals sternotomy wires, prosthetic aortic and mitral valves, and a greatly enlarged left atrium. Which symptom could most likely develop as a direct result of her left atrial enlargement? A. Nausea and vomiting B. Pain and tenderness over thoracic vertebral spinous processes C. Difficulty swallowing D. Epigastric pain after eating fatty foods E. Increased coughing

160 C. The left atrium lies directly anterior to the esophagus and compresses it when enlarged, result- ing in difficulty swallowing. Nausea and vomiting may be present but is a symptom of a wide array of dysfunctions and are not a direct result of the enlarged atrium. Pain and tenderness over the thoracic verte- bral spinous processes will not result because there is no compression of sensory nerves. Epigastric pain resulting from eating fatty foods is an indication of acute cholelithiasis, which is due to gallstones. Increased coughing could only result from irritation of the vagus nerves above the larynx, which is above the level of the left atria.

162 A 48-year-old man is admitted to the emergency department with chronic angina. Coronary angiography reveals nearly total blockage of the anterior interventricular artery just after it arises from the left coronary. In exposing this artery for a bypass procedure, which accompanying vein must be protected from injury? A. Middle cardiac B. Great cardiac C. Small cardiac D. Anterior cardiac E. Posterior cardiac

162 B. The great cardiac vein accompanies the anterior interventricular artery, and the middle accompanies the posterior interventricular artery. The small cardiac vein accompanies the right coronary artery as it wraps around the right coronary sulcus. This is the external demarcation of the atrioventricular septum and on the left contains the coronary sinus. On the anterior and posterior aspect of the right and left ventricles the anterior and posterior cardiac veins are found. The posterior accompany the circumflex branch of the left coronary artery and the anterior the small unnamed branches

165 A 55-year-old woman is to undergo a coronary bypass operation. The artery of primary concern is the vessel that arises from the circumflex artery in a left dominant heart. Which artery is this? A. Right marginal B. Anterior interventricular C. Left marginal D. Artery to the SA node E. Posterior interventricular

165 E. The question of dominance is determined by where the posterior interventricular artery arises. If it arises from the right coronary it is a right dominant heart; if from the circumflex of the left coronary it is left dominant. The anterior interventricular artery may anastomose with the posterior but is not a common origin or a determinant of dominance. The artery of the SA node and right marginal artery are branches of the right coronary artery and do not con- tribute to dominance.

167 A 42-year-old woman is admitted to the emergency department after a fall from the balcony of her apartment. During physical examination there is an absence of heart sounds, reduced systolic pressure, and engorged jugular veins. The condition that was created can be alleviated with which of the following procedures? A. Chest tube insertion superior to the rib B. Central venous line C. Nasogastric tube D. Thoracocentesis E. Pericardiocentesis

167 E. The patient exhibits signs of cardiac tam- ponade, known as Beck's triad: hypotension, muffled or absent heart sounds, and jugular-venous dis- tension. As a result, a pericardiocentesis procedure should be performed. Pericardiocentesis is a proce- dure whereby a needle is inserted into the pericardial space to extract fluid. Inserting a chest tube and tho- racocentesis are done to alleviate a pneumothorax and pleural effusion, respectively. A nasogastric tube will not alleviate any symptoms produced by pericardial fluid, and a central line would only provide access for intravenous fluids.

168 A 55-year-old man is admitted to the hospital with reduced heart rate and cardiac output. Which nerve fibers may have been damaged to cause these symptoms A. Preganglionic parasympathetics from the vagus nerve in the cardiac plexus B. Somatic efferents in the phrenic nerve C. Visceral afferents in the cardiopulmonary nerve D. Preganglionic sympathetics from T1 to T4 lateral horn E. T1 to T4 ventral horn neurons

168 D. The SA node receives information from the sympathetic preganglionic fibers T1 to T4 to increase the heart rate. If these fibers are damaged the pregan- glionic vagal fibers from the cardiac plexus are unop- posed and will slow down the heart rate. Somatic efferents in the phrenic nerve supply the diaphragm and have no effect on cardiac function. Visceral affer- ents in the cardiopulmonary nerve carry pain fibers from the heart and bronchi. T1 to T4 ventral horn neurons are somatic motor nerves and have no effect on cardiac function.

173 A 42-year-old woman delivered a male infant at term. On examination of the infant, vital signs were normal. Although he moved all limbs equally, there was generalized decreased muscle tone. His face was broad and flat with oblique eye fissures, flattened nose bridge, and protruding tongue from a small jaw. Auscultation of the chest revealed a systolic ejection murmur. Cardiac ultrasound showed a defect in the wall separating the right from left atria. The patient was diagnosed with trisomy 21 with an ASD. What is the most likely cause of the defect in this patient? A. Failed fusion of septum primum with the atrioventricular septum B. Failed fusion of septum primum with septum secundum C. Excess resorption of the cranial part of the septum primum D. Short septum secundum E. Incomplete resorption of the sinus venosus into the right atrium

173 A. There are four clinically significant types of ASD: ostium secundum defect, endocardial cushion defect with ostium primum defect, sinus venosus defect, and common atrium. The first two types of ASD are relatively common. Endocardial cushion defects with ostium primum occur in approximately 20% of persons with trisomy 21 (Down syndrome); otherwise, it is a relatively uncommon cardiac defect.

174 A 6.5 pound female infant was born to a 32-yearold woman by cesarean section. There was no cyanosis but a pansystolic murmur was auscultated. Cardiac ultrasound showed a small jet of blood directly between left and right ventricles during systole, through a defect in the cranial part of the interventricular septum. The infant was admitted to the Neonatal Intensive Care Unit and discharged on day 2. Which of the following is the most likely cause of the cardiac defect discovered in this patient? A. There is uneven partitioning of the bulbus cordis B. It causes a right to left shunt of blood at birth C. It is a cause of cyanosis at birth D. Failed fusion of the interventricular septum and endocardial cushions E. Failed closure of the septum primum and septum secundum at birth

174 D. VSDs are the most common type of coronary heart disease, accounting for approximately 25% of heart defects. A membranous VSD is the most common type but it may occur at any part of the interventriclar septum. A bulbus cordis partitioning defect may lead to persistent truncus arteriosus or transposition of the great vessels. VSD is an acyanotic heart disease. Septum primum and septum secundum may close after birth. Failure of closure will lead to patent foramen ovale.

175 A 30-year-old woman delivered a 6-lb female infant at term via spontaneous vaginal delivery. The infant was noted to have an elevated respiratory rate and was admitted to the neonatal intensive care unit. She developed a fever day later with no resolution of the tachypnea. A chest radiograph showed bilateral basal hazy opacification and a right-sided cardiac shadow. A whole-body CT scan was ordered which showed normal orientation of the other viscera. What is the most likely cause of this infant's cardiac condition? A. Posterior and superior growth of the primordial atrium B. Anterior and inferior growth of the primordial ventricle C. Anterior and inferior growth of the bulbus cordis D. Growth of primordial heart tube to the left E. Growth of the primordial heart tube to the right

175 D. With isolated dextrocardia, the abnormal position of the heart is not accompanied by displacement of other viscera. This defect is usually complicated by severe cardiac anomalies (e.g., single ventricle and transposition of the great vessels).

19 A 5-year-old boy has frequent episodes of fatigability and dyspnea. An ultrasound examination reveals an ASD, located at the opening of the superior vena cava. Which of the following types of ASDs are characteristic for this description? A. Ostium secundum B. Ostium primum C. Atrioventricular canal D. Common atrium E. Sinus venosus

19 E. Sinus venosus ASDs occur close to the entry of the superior vena cava in the superior portion of the interatrial septum. Ostium secundum ASDs are located near the fossa ovale and encompass both septum primum and septum secundum defects. An ostium primum defect is a less common form of ASD and is associated with endocardial cushion defects because the septum primum fails to fuse with the endocardial cushions, resulting in a patent foramen primum. An AV canal defect is not a clinically signifi- cant type of ASD. A common atrium is an uncommon type of ASD in which the interatrial septum is absent

21 A 2-day-old newborn male is admitted to the pediatric intensive care unit with cyanosis and tachypnea. Cardiac ultrasound (echocardiography) and magnetic resonance imaging (MRI) examinations reveal totally anomalous pulmonary connections. Which of the following embryologic events is responsible for this malformation? A. Abnormal septation of the sinus venosus B. Abnormal development of the septum secundum C. Abnormal development of the left sinus horn D. Abnormal development of the coronary sinus E. Abnormal development of common cardinal vein

21 A. The right horn of the sinus venosus has two divisions: One develops into the sinus venarum, the smooth interior aspect of the right atrial wall; the other half develops into the pulmonary veins.Abnormal septation of the sinus venosus can lead to inappropriate pulmonary connections. Abnormal development of the left sinus horn would present with abnormalities in the coronary sinus, whereas incor- rect development of the septum secundum can result in an ASD but would not be involved with anomalous pulmonary veins. The left sinusal horn develops into the coronary sinus, and the right sinusal horn is incor- porated into the right atrial wall. GAS 191-193, 203; N 211; McM 191-193

23 A 30-year-old man is diagnosed with a blockage of arterial flow in the proximal part of the thoracic aorta. Brachial arterial pressure is markedly increased, femoral artery pressure is decreased, and the femoral pulses are delayed. The patient shows no external signs of inflammation. Which of the following structures failed to develop normally? A. Second aortic arch B. Third aortic arch C. Fourth aortic arch D. Fifth aortic arch E. Ductus venosus

23 C. The fourth aortic arch develops into the aortic arch on the left side and the brachiocephalic and subclavian arteries on the right side of the embryo. Improper development of the arch of the aorta will cause an increased pressure in the subclavian artery and, subsequently, the brachial artery. Similarly, decreased flow through the aorta will lead to a decreased pressure in the femoral artery. The second aortic arch, specifically the dorsal aspect, develops into aspects of the small stapedial artery. The proximal part of the third aortic arch gives rise to the common carotid arteries, which supply the head. The fifth aortic arch is said not to usually develop in human embryos. The proximal part of the sixth aortic arch develops into the left pulmonary artery.

24 A 1-year-old child was admitted to the pediatric clinic due to severe dyspnea. An electrocardiogram (ECG) reveals cardiac arrhythmia and right ventricular hypertrophy. An angiogram reveals a PDA. From which of the following embryologic arterial structures does the PDA take origin? A. Left sixth aortic arch B. Right sixth aortic arch C. Left fifth aortic arch D. Right sixth aortic arch E. Left fourth aortic arch

24 A. The left sixth aortic arch is responsible for the development both of the pulmonary arteries and the ductus arteriosus. Without regression of the ductus arteriosus, a patent connection remains between aorta and the pulmonary trunk. The ductus arteriosus often reaches functional closure within 24 hours after birth, whereas anatomic closure and subsequent formation of the ligamentum arteriosum often occur by the twelfth postnatal week.

25 A 4-year-old girl is admitted to the hospital with high fever. Staphylococcus aureus is isolated from her blood cultures and antibiotic therapy is initiated. A loud, harsh murmur is heard on auscultation. A chest radiograph shows prominent pulmonary arteries. Echocardiography shows all the valves to be normal. Which congenital heart disease most likely explains these findings? A. Atrial septal defect B. Tetralogy of Fallot C. Coarctation of the aorta D. Patent ductus arteriosus E. Aortic atresia

25 D. With a PDA, an abnormal connection persists between the aorta and the pulmonary trunk. Blood leaving the left ventricle of the heart and into the aorta is reshunted back into the left pulmonary artery. This is responsible for the murmur heard during ausculta- tion of the heart. The diversion of blood to the pul- monary arteries causes increased atrial pressure, leading to enlarged, and therefore noticeable, pulmo- nary arteries on the chest radiograph. The tetralogy of Fallot often presents with a right-to-left shunt of blood flow through the ventricles. It is also associated with pulmonary artery stenosis, right ventricular hypertrophy, interventricular septal defect, and an overriding of the aorta. This condition would not present with a murmur, however. Atrial septal defects are often characterized by a left-to-right shunt of blood, which often presents with dyspnea and abnor- mal heart sounds. A chest radiograph would not reveal prominent pulmonary arteries in such cases. Both aortic atresia and coarctation of the aorta result in a narrowing of the aorta but would not lead to noticeable prominent pulmonary arteries on the radiograph.

26 A 3-day-old infant is admitted to the cardiology unit with severe cyanosis. During echocardiographic examination a right-to-left shunt is identified. Which of the following conditions will most likely produce this type of shunt? A. Interatrial septal defect B. Interventricular septal defect C. Patent ductus arteriosus D. Corrected transposition of the great arteries E. Common truncus arteriosus

26 E. A common truncus arteriosus results from failure of separation of the pulmonary trunk and aorta. Without proper perfusion of the child by oxy- genated blood, severe cyanosis will result.

50 A 54-year-old man is admitted to the hospital with dyspnea. Physical examination and echocardiographic studies reveal severe mitral valve prolapse. Auscultation of this valve is best performed at which location? A. Left fifth intercostal space, just below the nipple B. Right lower part of the body of the sternum C. Right second intercostal space near the lateral border of the sternum D. Directly over the middle of the manubrium E. Left second intercostal space near the lateral border of the sternum

50 A. The left fifth intercostal space, just below the left nipple, is typically the best location to listen to the mitral valve. Although the mitral valve is located at the fourth intercostal space just to the left of the sternum, the sound is best realized "downstream" from the valve. The right lower part of the body of the sternum is the location of the tricuspid valve. The right second intercostal space near the lateral border of the sternum is the typical location of auscultation of the aortic valve. It is difficult to hear valvular sounds through bone, so auscultating directly over the middle of the manubrium is not a good choice. The left second intercostal space near the lateral border of the sternum is the site chosen typically for ausculta- tion of the pulmonary valve

51 A 48-year-old male patient is admitted with chronic angina. Coronary angiography reveals nearly total blockage of the circumflex artery near its origin from the left coronary artery. When this artery is exposed to perform a bypass procedure, what accompanying vein must be protected from injury? A. Middle cardiac B. Great cardiac C. Small cardiac D. Anterior cardiac E. Posterior cardiac

51 B. The great cardiac vein (anterior interventricular vein) takes a pathway initially beside the anterior interventricular coronary artery (left anterior

52 A 55-year-old patient is to undergo a coronary bypass operation. The artery of primary concern is the vessel that supplies much of the left ventricle and the right and left bundle branches of the cardiac conduction system. Which artery is the surgeon most concerned with? A. Right marginal B. Anterior interventricular C. Circumflex D. Artery to the sinu-atrial (SA) node E. Posterior interventricular

52 B. The anterior interventricular artery (left anterior interventricular artery, LAD) supplies the right and left ventricles and anterior two thirds of the IV septum. The right marginal artery supplies the right ventricle and apex of the heart; therefore, it does not supply the left ventricle. The left coronary circumflex artery supplies the left atrium and left ventricle; it courses posteriorly in, or near to, the coronary sulcus and supplies the posterior portion of the left ventricle and left atrium. The artery to the SA node is a branch of the right coronary artery and does not supply the left ventricle. The posterior interventricular (posterior descending) artery arises from the right coronary artery in about 67% of people (this is referred to as a right dominant pattern) and supplies the posterior aspect of both ventricles and the posterior third of the interventricular septum

53 A 58-year-old male patient presents himself to the emergency department with severe angina. Upon cardiac catheterization, it is found that he has a significant occlusion in his right coronary artery, just distal to the right sinus of the aortic valve. His collateral cardiac circulation is minimal. Assuming the patient is right coronary dominant, which of the following arteries would be most likely to still have normal blood flow? A. Right (acute) marginal artery B. Atrioventricular nodal artery C. Posterior interventricular artery D. SA nodal artery E. Anterior interventricular artery

53 E. The anterior interventricular artery (LAD) arises from the left coronary artery. If there is occlu- sion in the right coronary artery, the anterior inter- ventricular artery will still have normal blood flow. The right marginal artery branches from the right coronary artery; therefore, if there is occlusion of the right coronary artery, flow from the marginal artery will be compromised. The AV nodal artery is supplied by the coronary artery that crosses the crux of the heart posteriorly. If this artery arises from the right coronary, supply to the AV node might be reduced, depending upon collateral supply. The SA nodal artery is supplied by the right coronary artery in about 55% of the population (only 35% from the left); as the patient is right coronary dominant, it would be pre- dicted that the SA nodal artery will not have normal blood flow.

54 A 55-year-old man is admitted to the emergency department with severe chest pain. Coronary angiography reveals that the patient's right coronary artery is free of pathology. The left coronary artery is found to be 70% to 80% occluded at three points proximal to its bifurcation into the circumflex and left anterior descending arteries. Having a left dominant coronary circulation, and without surgery, what is the most likely explanation for a poor prognosis for recovery of this patient to a normally active life? A. All the branches of the coronary artery are end arteries, precluding the chance that anastomotic connections will occur. B. It is probable that the anterior and posterior papillary muscles of the tricuspid valve have been damaged. C. The blood supply of the SA node is inadequate. D. The development of effective collateral circulation between anterior and posterior interventricular arteries will not be possible. E. The blood supply of the atrioventricular (AV) node will be inadequate.

54 D. Because the patient is left coronary artery dominant, if there is 70% to 80% occlusion of the left coronary, there will be deficiencies in flow both in the anterior descending and circumflex coronary arteries. No possibility is available for collateral flow from the posterior descending interventricular artery, for it too would be derived from the left coronary, by way of the circumflex artery. If the patient does not undergo surgery to remove or bypass the occlusion, he will be unable to have any substantial type of collateral cir- culation between the two major branches of the left coronary. The branches of the coronary arteries are not end arteries, and there are anastomoses between them. The papillary muscles of the tricuspid valve would not be affected with left coronary artery occlu- sion. The blood supply to the SA node would not be inadequate. The blood supply of the region of the AV node might or might not be adequate, for it could still be supplied by a branch of the right coronary artery.

55 A 35-year-old woman is admitted to the hospital with dyspnea. During physical examination her S1 heart sound is very loud. Which of the following valves is most likely defective? A. Mitral valve B. Aortic C. Pulmonary D. Aortic and pulmonary E. Tricuspid

55 A. The mitral valve corresponds to the S1 heart sound produced during systole. The aortic and pulmonary valves correspond to the S2 heart sound produced during diastole. The tricuspid valve also corresponds with the S1 heart sound. The aortic valve, however, corresponds with the S2 sound, so this answer would be incorrect.

56 A 72-year-old man is admitted to the hospital with severe chest pain. ECG examination provides evidence of severe myocardial infarction of the lower part of the muscular interventricular septum. The function of which of the following valves will be most severely affected? A. Pulmonary B. Aortic C. Tricuspid D. Mitral E. Eustachian

56 C. The interventricular septum is intimately involved with the tricuspid valve on the right side, via the muscular connections of the septomarginal tra- beculum (moderator band) to the anterior papillary muscle. Therefore, if the electrical system of the heart is disrupted, as with a myocardial infarction in the upper portion of the muscular septum, the innerva- tion of the interventricular septum will be compro- mised and the tricuspid valve will be directly affected. None of the other valves is directly involved with the interventricular septum.

58 A 3-month-old infant is diagnosed with a membranous VSD. A cardiac operation is performed, and the septal defect is patched inferior to the noncoronary cusp of the aorta. Two days postoperatively, the infant develops severe arrhythmias affecting both ventricles. Which part of the conduction tissue was most likely injured during the procedure? A. Right bundle branch B. Left bundle branch C. Bundle of His D. Posterior internodal pathway E. Atrioventricular node

58 C. The atrioventricular bundle (of His) is a col- lection of specialized cardiac muscle cells (Purkinje fibers) that carry electrical activity to the right and left bundle branches. Because both ventricles are affected, this is the logical site of injury, for this bundle leads to the bundle branches supplying both ventricles. An injury either to the right or left bundle branch would affect only one ventricle. Terminal Pur- kinje fibers transmit the electrical activity to the greater sections of the ventricles, yet dysfunction in the terminal part of the conduction system would affect only a small section of one ventricle, not both. The atrioventricular node is a group of specialized cardiac muscle cells that serve to decrease the rate of conduction to the ventricles and is located in the region deep to the septal wall of the right atrium. The posterior internodal pathway is in the roof of the right atrium and is not involved here.

59 A 62-year-old man was admitted to the hospital with intense left chest pain. ECG and echocardiography reveal myocardial infarction and pulmonary valve regurgitation. Emergency coronary angiography is performed and provides evidence that the artery supplying the upper portion of the anterior right ventricular free wall is occluded. Which of the following arteries is most likely to be occluded? A. Circumflex B. Anterior interventricular artery C. Posterior interventricular artery D. Artery of the conus E. Acute marginal branch of the right coronary artery

59 D. The artery of the conus is given off from the right coronary artery and winds around the conus arteriosus. The conus region is the superior part of the right ventricle that tapers into a cone (infundibu- lum) where the pulmonary valve leads into the pul- monary trunk. This conus artery supplies the upper portion of the anterior right ventricle and usually has a small anastomotic connection with the anterior interventricular (left anterior descending) branch of the left coronary artery. The circumflex artery supplies the left atrium and ventricle and does not supply the right ventricle except when the posterior inter- ventricular (posterior descending) artery arises from the circumflex, or in unusual cases in which the cir- cumflex passes to the surface of the right ventricle. The anterior interventricular artery supplies the right and left ventricles and the anterior two thirds of the IV septum. It is given off by the left coronary artery and does not specifically supply the upper portion of the right ventricle. The posterior interventricular artery supplies the right and left ventricles and the posterior third of the IV septum. It does not supply the upper portion of the right ventricle.

60 A 3-month-old male infant died unexpectedly in his sleep. The pathologist examined the histologic slides of tissue samples taken from the heart of the infant and observed that a portion of the conduction tissue that penetrates the right fibrous trigone had become necrotic. As a result, a fatal arrhythmia probably developed, leading to the death of the infant. Which of the following parts of the conduction tissue was most likely interrupted? A. Right bundle branch B. The bundle of Bachmann C. The left bundle branch D. The atrioventricular bundle of His E. The posterior internodal pathway

60 D. The atrioventricular bundle of His is a strand of specialized cardiac muscle fibers (Purkinje fibers) that arises from the atrioventricular node and passes through the right fibrous trigone. The right fibrous trigone (central fibrous body) is a dense area of con- nective tissue that interconnects the mitral, tricuspid, and aortic valve rings. After reaching the upper portion of the muscular interventricular septum, the bundle of His splits into right and left bundle branches. The bundle of Bachmann is a collection of fibers running from the SA node to the left atrium and is the only collection of conducting fibers to innervate the left atrium. Finally, the posterior internodal pathway, also known as Thorel's pathway, is the prin- cipal pathway of electrical activation between the SA node and atrioventricular node in humans.

61 A 42-year-old woman is admitted to the hospital after blunt trauma to her sternum by the steering wheel during a car crash. Ultrasound examination reveals a cardiac tamponade. ECG data indicate that the heart has been severely injured. Which of the following cardiac structures will most likely be injured? A. Right ventricle B. Obtuse margin of the left ventricle C. Right atrium D. Left atrium E. Apex of the left ventricle

61 A. The sternocostal surface of the heart consists mostly of the right ventricle. Therefore, an anterior injury to the thorax would mostly likely first affect the right ventricle because it is adjacent to the deep surface of the sternum. However, if the question did not ask which part of the heart has been injured but which part of the heart will most likely be compressed by the cardiac tamponade, the correct answer would have been the right atrium. This is due to the fact that the right atrium has lower pressures than the other cardiac components.

62 A 69-year-old man is admitted to the hospital with intense left chest pain. ECG reveals hypokinetic ventricular septal muscle, myocardial infarction in the anterior two thirds of the interventricular septum and left anterior ventricular wall. The patient's ECG also exhibited left bundle branch block. Which of the following arteries is most likely occluded? A. Circumflex B. Proximal right coronary C. Proximal left coronary D. Proximal left anterior interventricular artery E. Posterior interventricular artery

62 D. The tissues affected in this case, the interven- tricular septum and anterior ventricular wall, are mostly supplied by the proximal portion of the left anterior interventricular artery. If the circumflex artery were blocked, the left atrium and left ventricle would be affected (in a right coronary dominant heart). If the right coronary artery were occluded, again assum- ing right coronary dominance, it would affect the right atrium, the SA and atrioventricular nodes, part of the posterior left ventricle, and the posterior part of the interventricular septum. If the left coronary artery (LCA) were blocked, most of the left atrium and left ventricle, the anterior two thirds of the interventricu- lar septum, and the area of bifurcation of the bundle of His would be affected. If the posterior interven- tricular artery were occluded, it would affect the right and left ventricles and the posterior third of the inter- ventricular septum. The circumflex and the anterior interventricular arteries are branches of the LCA, and the posterior interventricular artery is most commonly a branch of the terminal segment of the right coronary artery.

63 A 49-year-old woman is admitted to the hospital complaining of severe, crushing, retrosternal pain during the preceding hour. An ECG reveals that she is suffering from acute myocardial infarction in the posterior aspect of her left ventricle and posteromedial papillary muscle. A coronary angiogram is performed and the patient is found to have left dominant coronary circulation. Which of the following arteries is the most likely to be occluded? A. Artery of the conus B. Right coronary artery C. Circumflex D. Right acute marginal E. Left diagonal

63 C. A "left coronary dominant" circulation means, put simply, that the left coronary artery (LCA) pro- vides the posterior interventricular artery as a termi- nal branch of the coronary circumflex. The posterior aspect of the heart is composed primarily of the left ventricle and is supplied by the posterior interven- tricular branch. The artery of the conus supplies the right ventricular free wall. If the right coronary artery were occluded (in a right coronary dominant heart), it would affect the right atrium, right ventricle, the SA and atrioventricular nodes, the posterior part of the interventricular septum, and part of the posterior aspect of the left ventricle. The right acute marginal artery supplies the inferior margin of the right ven- tricle. The left diagonal arteries arise most commonly from the anterior interventricular (left-anterior descending) artery but can also arise as branches of the left coronary or the circumflex.

65 A 4-year-old boy is operated on for a correction of a small, muscular interventricular septal defect. To access the right side of the interventricular septum, a wide incision is first made in the anterior surface of the right atrium. Instruments are then inserted through the tricuspid valve to correct the VSD. Which of the following structures is the most crucial to protect during the opening of the right atrium? A. Crista terminalis B. Pectinate muscles C. Tricuspid valve D. Eustachian valve E. Coronary sinus

65 A. The crista terminalis is a muscular ridge that runs from the opening of the superior vena cava to the inferior vena cava. This ridge provides the path taken by the posterior internodal pathway (of Thorel) between the SA and atrioventricular nodes. The crista also provides the origin of the pectinate muscles of the right auricle. The tricuspid valve is located below the inferior vena cava, between the right atrium and right ventricle. The eustachian valve is an embryo- logic remnant of the valve of the inferior vena cava. The ostium of the coronary sinus is located between the right atrioventricular orifice and the inferior vena cava.

67 A 55-year-old man is brought to the emergency department after his motorcycle collided with an automobile. He is hypotensive, his pulse is irregular, and he shows other signs of substantial blood loss. MRI and CT scan evaluations reveal profuse abdominal bleeding. A decision is made to enter the chest so that the descending thoracic aorta can be clamped to minimize blood loss and to preserve cerebral blood flow. After surgical entrance into the thorax, the fibrous pericardium is elevated with forceps and punctured. A midline, longitudinal incision of the pericardium would best be made to prevent injury to which of the following structures? A. Auricular appendage of the left atrium B. Coronary sinus C. Left anterior descending artery D. Left phrenic nerve E. Left sympathetic trunk

67 C. The anterior interventricular (left anterior descending) artery lies anteriorly and to the left and descends vertically to the left toward the apex. It can be more easily injured by a transverse incision of the pericardium, which would cross perpendicular to this artery. The auricular appendage of the left atrium is located posteriorly; therefore, it would not be injured in an anterior longitudinal incision. The coronary sinus is between the right atrioventricular orifice and the inferior vena cava and would not be affected. The left phrenic nerve lies between the heart and the left lung and is too deep to be injured in this incision. The left sympathetic trunk is also too posterior to be injured.

68 During cardiac surgery of a 45-year-old man, the cardiac surgeon can place her fingers in the transverse pericardial sinus, if necessary. This allows the surgeon to easily place a vascular clamp upon which of the following vessels? A. Right and left pulmonary veins B. Superior and inferior vena cava C. Right and left coronary arteries D. Pulmonary trunk and ascending aorta E. Pulmonary trunk and superior vena cava

68 D. A finger passing through the transverse peri- cardial sinus passes directly behind the two great arteries exiting the heart, allowing the surgeon to rather easily place a vascular clamp upon the pulmo- nary trunk and ascending aorta. The other vessels listed are not readily accessible by way of the trans- verse sinus.

69 A 48-year-old male patient is scheduled to have a coronary arterial bypass because of chronic angina. Coronary arteriography reveals nearly total blockage of the posterior descending interventricular artery. In exposing this artery to perform the bypass procedure, which accompanying vessel is most susceptible to injury? A. Middle cardiac vein B. Great cardiac vein C. Small cardiac vein D. Anterior cardiac vein E. Coronary sinus

69 A. The middle cardiac veins run parallel with the posterior interventricular (posterior descending) artery and drains directly into the coronary sinus. The great cardiac vein parallels the anterior interventricu- lar artery and the small cardiac veins pass parallel with the right marginal artery. The anterior cardiac veins are several small veins that drain directly into the right atrium. The coronary sinus is a wide venous channel that runs from left to right in the posterior part of the coronary groove.

70 A 54-year-old man is admitted to the hospital with severe chest pain. ECG examination reveals a myocardial infarction. If the posterior interventricular branch in the patient arises from the right coronary artery, which part of the myocardium will most likely have its blood supply reduced if the circumflex branch of the left coronary artery becomes occluded from an atherosclerotic plaque? A. Anterior part of the interventricular septum B. Diaphragmatic surface of the right ventricle C. Infundibulum D. Lateral wall of the left ventricle E. Posterior part of the interventricular septum

70 D. The left coronary artery bifurcates into the anterior interventricular artery (left anterior descend- ing: LAD) and the coronary circumflex branch. The circumflex branch gives off the left marginal branch, which supplies the lateral wall (obtuse margin) of the left ventricle. The anterior part of the interventricular septum is supplied by the LAD. The diaphragmatic surface of the right ventricle is supplied by the posterior descending artery and the right marginal, a branch of the right coronary artery. The infundibu- lum, also known as the conus arteriosus, is the outflow portion of the right ventricle. The posterior part of the interventricular septum is supplied by the posterior descending artery, in most cases a branch of the right coronary artery.

71 A 70-year-old man with a history of two previous myocardial infarctions is admitted to the hospital with severe chest pain. ECG reveals a new myocardial infarction and ventricular arrhythmia. Coronary angiography reveals that the right coronary artery is blocked just distal to the origin of the right marginal artery in a right coronary dominant circulation. Which of the following structures would most likely be affected after such a blockade? A. Right atrium B. SA node C. AV node D. Lateral wall of the left ventricle E. Anterior interventricular septum

71 C. The atrioventricular (AV) node is most com- monly supplied by a branch of the right coronary artery. This branch arises at the crux of the heart (the point of junction of all four cardiac chambers posteriorly); this is the location of the occlusion. The right atrium is supplied by the right coronary artery, which additionally supplies the SA node. The left marginal artery supplies the lateral wall of the left ventricle. The anterior portion of the interventricular septum is supplied by the anterior interventricular artery.

72 A 43-year-old woman is diagnosed with mitral valve stenosis. During physical examination the first heart sound is abnormally loud. Which of the following heart valves are responsible for the production of the first heart sound? A. Aortic and mitral B. Aortic and tricuspid C. Tricuspid and mitral D. Mitral and pulmonary E. Tricuspid and pulmonary

72 C. The first heart sound is caused by the closure of the tricuspid and mitral valves. The second heart sound is caused by the closure of the aortic and pul- monary valves.

73 A 75-year-old woman is admitted to the hospital with anginal pain. ECG reveals myocardial infarction and a right bundle branch block. During physical examination the patient has a loud second heart sound. Which of the following heart valves are responsible for the production of the second heart sound? A. Aortic and pulmonary B. Aortic and tricuspid C. Tricuspid and mitral D. Mitral and pulmonary E. Tricuspid and pulmonary

73 A. The second heart sound is caused by the closure of the aortic and pulmonary valves. The first sound by the heart is caused by the closure of the tricuspid and mitral valves.

74 Ten days after a surgical procedure to correct her cardiac malformation, a 3-month-old infant died unexpectedly in her sleep. After an autopsy, the pathologist reported as follows: "A significant portion of the conduction tissue was found to be necrotic. The area of the necrotic tissue was located inferior to the central fibrous body, membranous septum, and septal leaflet of the tricuspid valve. Further examination revealed infarction of the surrounding tissue. The rest of the heart was unremarkable." Which of the following arteries was most likely occluded? A. Artery of the conus B. SA node artery C. AV node artery D. First septal perforator of the anterior interventricular artery E. All of the above

74 D. The first septal perforating branch of the anterior interventricular artery (left anterior descend- ing; LAD) is the first branch of the LAD that supplies the conducting tissue of the heart; it passes directly to the point of bifurcation of the common atrioven- tricular bundle of His. The other vessels listed have no anatomic relation to the area of ischemia.

75 A 55-year-old man is undergoing an aortic valve replacement. During the procedure the heart is connected to the heart lung machine. As the surgeon explores the oblique pericardial sinus, which of the following is not directly palpable with the tips of the fingers? A. Inferior vena cava B. Superior vena cava C. Posterior wall of the left atrium D. Inferior right pulmonary vein E. Right atrium

75 B. The superior vena cava empties into the right atrium on the superior aspect of the heart; it is not directly palpable from the oblique sinus. The oblique sinus is a cul-de-sac providing access to the inferior vena cava, the posterior wall of the left atrium, right atrium, and the right and left pulmonary veins.

76 A 42-year-old woman is admitted urgently to the emergency department after suffering a penetrating wound to her chest from an ice pick during a violent domestic dispute. Physical and ultrasound examinations reveal that the patient has cardiac tamponade. Which of the following will most likely be found during physical examination? A. There will be a visible or palpable decrease in the dimensions of the external jugular and internal jugular vein B. There will be gradual enlargement of the ventricles in diastole C. The difference between systolic and diastolic arterial pressures will increase significantly D. There will be diminished heart sounds E. The pulses in the internal carotid arteries will become increasingly distinct, as detected behind the angles of the mandible

76 D. Cardiac tamponade is characterized by hypo- tension, tachycardia, muffled heart sounds, and jugular vein distention. Bleeding into the pericardial cavity would muffle the heart sounds because of the increased distance between the chest wall and the heart, leading to "distant" heart sounds. When the effusion is particularly severe, the heart may take on a "water bottle" appearance on an anterior-posterior radiograph.

77 During surgical repair of a congenital cardiac anomaly in a 15-year-old boy with a right dominant coronary arterial system, the surgeon accidentally injured a vessel that usually supplies part of the conduction system. This results in intermittent periods of atrioventricular block and severe arrhythmia. The injured artery was most likely a direct branch of which of the following arteries? A. Distal anterior interventricular artery B. Circumflex artery C. Left coronary artery D. Marginal artery E. Right coronary artery

77 E. "Right coronary dominant circulation" refers simply to the fact that the right coronary artery pro- vides origin for the posterior interventricular (poste- rior descending) coronary artery. In such cases, it provides supply for the SA and atrioventricular nodes. It might be anticipated that right coronary blockage could result in dysfunction of the atrioventricular node, if collateral supply is poor or absent. The LAD, circumflex, and left marginal are all branches of the left coronary artery. The right marginal artery mar- ginal is a branch of the right coronary artery.

78 A 42-year-old woman is admitted to the hospital with dyspnea. Imaging reveals severe mitral valve regurgitation. Which of the following structures prevents regurgitation of the mitral valve cusps into the left atrium during systole? A. Crista terminalis B. Crista supraventricularis C. Pectinate muscles D. Chordae tendineae E. Trabeculae carneae

78 D. The chordae tendineae are fibrous cords that connect papillary muscles to valve leaflets. The restraint provided by these cords on the valve leaflets (along with contraction of the papillary muscles) pre- vents the prolapse of the mitral valve cusps into the left atrium. The crista terminalis is a ridge that runs from the opening of the inferior vena cava to the superior vena cava. Trabeculae carneae are irregular ridges of myocardium that are present within the ventricles.

79 A 58-year-old woman with cardiac arrhythmia has undergone a procedure to implant a pacemaker. The electrical conducting leads for the pacemaker must be passed into the heart from the pacemaker. Which of the following is the correct order of structures for passage of the leads into the right ventricle? A. Brachiocephalic vein, superior vena cava, mitral valve, right ventricle B. Superior vena cava, right atrium, mitral valve, right ventricle C. Superior vena cava, right atrium, tricuspid valve, right ventricle D. Brachiocephalic vein, superior vena cava, right atrium, tricuspid valve, right ventricle E. Brachiocephalic vein, superior vena cava, right atrium, mitral valve, right ventricle

79 D. The correct path that leads to the right ven- tricle for the lead of the pacemaker is the brachioce- phalic vein (could be right or left; pacemakers are more commonly placed on the left in which case it would be the left brachiocephalic vein), superior vena cava, right atrium, tricuspid valve, and right ventricle.

80 A 68-year-old man patient in the cardiology ward complains at each mealtime of difficulty in swallowing (dysphagia). Radiologic studies reveal significant cardiac hypertrophy. A barium swallow, followed by radiologic examination of the thorax, reveals esophageal constriction directly posterior to the heart. Which of the following is the most likely cause of the patient's dysphagia? A. Mitral valve stenosis B. Pulmonary valve stenosis C. Regurgitation of the aorta D. Occlusion of the anterior interventricular artery E. Occlusion of the posterior interventricular artery

80 A. Mitral stenosis leads to left atrial dilation, which can exert a compressive effect on the esopha- gus. The pulmonary valve is located between the outflow tract of the right ventricle and the pulmonary trunk. The aortic valve is located between the left ventricle and the aorta. Anterior interventricular (LAD) and posterior interventricular (posterior de- scending) arterial occlusions can cause a myocardial infarction, but not dysphagia. In the normal position of the heart the left atrium lies most posteriorly. Therefore, a stenosis of the mitral valve (atrioven- tricular valve between left atrium and left ventricle) would lead to enlargement of the left atrium, which would in turn impinge upon the esophagus. A steno- sis of the pulmonary valve would have no effect upon the esophagus because of the anterior position of the pulmonary trunk in the thorax. Regurgitation through any valve will ultimately decrease systemic blood flow. An occlusion of a coronary artery will lead to ischemia and possibly myocardial infarction.

81 A 35-year-old woman is admitted to the emergency department because of cardiac arrhythmia. ECG examination reveals that the patient suffers from atrial fibrillation. Where is the mass of specialized conducting tissue that initiates the cardiac cycle located? A. At the junction of the coronary sinus and the right atrium B. At the junction of the inferior vena cava and the right atrium C. At the junction of the superior vena cava and the right atrium D. Between the left and right atria E. In the interventricular septum

81 C. The SA node, the primary pacemaker of the heart, is a mass of specialized cardiac cells within the myocardium at the upper end of the crista terminalis, near the opening of the superior vena cava into the right atrium. The AV node is at the junction of the coronary sinus and the right atrium upon the right fibrous trigone (central fibrous body). The eustachian valve directs blood from the inferior vena cava and through the right atrium toward the tricuspid valve ostium. The interatrial septum is located between the left and right atria. The septomarginal trabeculum (moderator band) arises from the muscular portion of the interventricular septum and passes to the base of the anterior papillary muscle in the right ventricle. The moderator band carries the right bundle branch of the conduction system just beneath its endocardial layer.

82 A 45-year-old woman is admitted to the hospital with swelling (edema) of the lower limbs. Ultrasound examination reveals an incompetent tricuspid valve. Into which area will regurgitation of blood occur in this patient? A. Pulmonary trunk B. Left atrium C. Ascending aorta D. Right atrium E. Left ventricle

82 D. The tricuspid valve is the atrioventricular valve located between the right atrium and right ven- tricle. An incompetent valve would allow blood to regurgitate into the right atrium during systole and subsequently raise pressure in the venous system, increasing capillary pressure and causing edema. A regurgitation of blood into the pulmonary trunk would be a result of an incompetent pulmonary valve. Regur- gitation of blood from the left ventricle back into the left atrium is a result of prolapse of the mitral valve. There is no direct anatomic relationship between the tricuspid valve and the ascending aorta. Blood would pool in the left ventricle in the event of aortic valve incompetence.

83 A 34-year-old man with a complaint of sharp, localized pain over the thoracic wall is diagnosed with pleural effusion. Through which intercostal space along the midaxillary line is it most appropriate to insert a chest tube to drain the effusion fluid? A. Fourth B. Sixth C. Eighth D. Tenth E. Twelfth

83 C. To avoid damaging the lungs, a chest tube should be placed below the level of the lungs, in the costodiaphragmatic recess. Such a point of entrance for the tube would be the eighth or ninth intercostal space. At the midclavicular line, the costodiaphrag- matic recess is localized between intercostal spaces 6 and 8, at the midaxillary line between 8 and 10, and at the paravertebral line between ribs 10 and 12.

84 A 51-year-old man is admitted to the hospital with severe dyspnea. Radiologic examination reveals a tension pneumothorax. Adequate local anesthesia of the chest wall prior to insertion of a chest tube is necessary for pain control. Of the following layers, which is the deepest that must be infiltrated with the local anesthetic to achieve adequate anesthesia? A. Endothoracic fascia B. Intercostal muscles C. Parietal pleura D. Subcutaneous fat E. Visceral pleura

84 C. The parietal pleura is innervated by the inter- costal nerves and is very sensitive to pain, in this case being somatic innervation. Therefore, the parietal pleura is the deepest layer that must be anesthetized to reduce pain during aspiration or chest tube placement.

85 A 5-year-old boy had been playing with a small toy car. Soon after, he put a wheel from one of the cars in his mouth and began choking and coughing. Where in the tracheobronchial tree is the most common site for a foreign object to lodge? A. The right primary bronchus B. The left primary bronchus C. The carina of the trachea D. The beginning of the trachea E. The left tertiary bronchus

85 A. The right main bronchus is the shorter, wider, and more vertical primary bronchus. Therefore, this is most often the location that foreign objects will likely be lodged. The left primary bronchus is not as vertical and therefore does not present the path of least resistance. (It must be understood, however, that in some cases of aspiration, the foreign body can pass into the left primary bronchus rather than the right bronchus!) The carina is a ridge separating the open- ings of left and right bronchi, the "fork in the road," so to speak. The trachea is a tubular structure sup- ported by incomplete cartilaginous rings, and the like- lihood that an object will be lodged there is minimal. It is unlikely that a foreign object would descend so far as to obstruct a tertiary bronchus, although this could happen.

92 An 18-year-old man is admitted to the emergency department because of a significant nose bleed and a headache that has worsened over several days. He also complains of fatigue. Upon examination it is noted that brachial artery pressure is markedly increased, femoral pressure is decreased, and the femoral pulses are delayed. The patient shows no external signs of inflammation. Which of the following is the most likely diagnosis? A. Coarctation of the aorta B. Cor pulmonale C. Dissecting aneurysm of the right common iliac artery D. Obstruction of the superior vena cava E. Pulmonary embolism

92 A. Increased arterial pressure in the upper limbs (as demonstrated in the brachial artery) and decreased pressure in the lower limbs (as demonstrated in the femoral artery) are common symptoms of coarctation of the aorta. Other symptoms include tortuous and enlarged blood vessels above the coarctation and an increased risk of cerebral hemorrhage. This condition of coarctation occurs when the aorta is abnormally constricted during development. The patient does not complain of respiratory distress, so cor pulmonale would not likely be the underlying condition. Dissec- tion of the right common iliac artery would not result in nosebleed or headache. Obstruction of the superior vena cava would not account for decreased femoral pulse. A pulmonary embolism will not present with these symptoms.

93 A 22-year-old man is diagnosed with signs of reduced aortic flow. Upon examination it is noted that brachial artery pressure is markedly increased, femoral pressure is decreased, and the femoral pulses are delayed. The patient shows no external signs of inflammation. Which of the following conditions will most likely be observed in a radiologic examination? A. Flail chest B. Pneumothorax C. Hydrothorax D. Notching of the ribs E. Mediastinal shift

93 D. The diagnosis for these symptoms is coarctation of the aorta. This condition occurs when the aorta is abnormally constricted. One of the cardinal radiographic signs is a characteristic rib notching. "Notching" of the ribs is due to the reversal of direction of blood flow through the anterior intercostal branches of the internal thoracic artery, as these usually small arteries carry collateral arterial blood flow to the lower thoracic portion of the aorta inferior to the coarctation. Enlargement and vibration of the intercostal arteries against the rib results in erosion ("notching") of the subcostal grooves, which is visible on radiography.

Postoperative examination of a 68-year-old man who underwent mitral valve replacement demonstrates significant cardiac hypertrophy (Fig. 2-5). Which of the following structures would be most likely compressed? A. Esophagus B. Pulmonary trunk C. Superior vena cava D. Trachea E. Inferior vena cava

A. Cardiac hypertrophy is a compensatory mechanism of the myocardium in response to increasing demands on the heart due to ischemia, incompetent valves, or hypertension. The increased size of the heart muscle would most likely compress the esophagus, and due to the incompetent mitral valve, a back- flow of blood into the left atrium can cause a left atrial dilation. The left atrium lies just anteriorly to the esophagus in the mediastinum. The pulmonary trunk is located superiorly and delivers blood to the lungs, so cardiac hypertrophy would not cause direct compression to this structure. The superior vena cava and inferior vena cava are vessels that deliver blood to the right atrium and are not likely to be compressed in this example of cardiac hypertrophy. The heart is inferior to the trachea.

A cyanotic 9-year-old boy is brought to the emergency department, coughing up blood and complaining of severe chest pain. Examination by the physician reveals a heart arrhythmia and a heart murmur. Looking at the patient's history, the physician diagnoses the boy with Eisenmenger's syndrome, a left-to-right shunt converted into a right-to-left shunt secondary to elevated pulmonary artery pressure. Which of the following disorders could be the initial congenital defect (left-toright shunting) causing this syndrome? A. Ventricular septal defect B. Ebstein's anomaly C. Underdeveloped left ventricle (hypoplastic left heart syndrome) D. Common atrioventricular canal E. Large foramen secundum

A. Eisenmenger's syndrome or tardive cyanosis is the process in which a left-to-right shunt caused by a congenital heart defect causes increased flow through the pulmonary vasculature, causing pulmonary hypertension. This in turn causes increased pressure in the right side of the heart and reversal of the shunt into a right-to-left shunt. Eisenmenger's syndrome is a cyanotic heart defect characterized by a long-standing intracardiac shunt caused commonly by ventricular septal defects. In Ebstein's anomaly the septal leaflet of the tricuspid valve is displaced toward the apex of the right ventricle of the heart. There is subsequent atrialization of part of the right ventricle (which is now contiguous with the right atrium). This causes the right atrium to enlarge and the anatomic right ventricle to be small. Here there is an initial right-to-left shunt that results in a cyanotic baby. With the underdeveloped left ventricle, both the aorta and left ventricle are underdeveloped before birth, and the aortic and mitral valves are each too small to allow sufficient blood flow. As blood returns from the lungs to the left atrium, it must pass through an atrial septal defect to the right side of the heart. In this defect babies appear cyanotic. In the common atrioventricular canal defect the heart has one common chamber due to defects in the formation of its septae. Patients develop pulmonary hypertension by the second year of life. In a large foramen secundum (foramen ovale) the atrial septal defect (ASD) does lead to pulmonary hypertension but has a much slower progression. Symptoms usually appear after the third decade of life.

A 2-day-old infant is diagnosed with transposition of the great arteries. If this condition were to be left untreated for more than 4 months, it would be fatal. Which of the following structures must remain patent so that the infant can survive until surgical correction of the malformation? A. Ductus arteriosus B. Umbilical arteries C. Umbilical vein D. Coarctation of the aorta E. Pulmonary artery stenosis

A. In a case of transposition of the great arteries, oxygenated blood travels from the left ventricle into the pulmonary trunk, where it will eventually reach the lungs. In contrast, the aorta would be carrying deoxygenated blood into the systemic circulation. A PDA acts as a shunt between the aorta and pulmonary trunk, allowing oxygenated and deoxygenated blood to mix and therefore allowing some oxygenated blood to reach the tissues. None of the other answer choices would be correct for this problem; with these struc- tures remaining patent, the body would still not receive sufficient oxygenated blood for survival to be possible.

Oxygenated blood that reaches the heart in fetal circulation from the IVC passes through which of the following valves formed by a portion of the septum secundum? A. Foramen ovale B. Ductus arteriosus C. Foramen primum D. Ductus venosus E. Truncus arteriosus

A. In the fetal heart the left and right atria com- municate with each other by an opening in the septum secundum, referred to as the foramen ovale. Ductus arteriosus is a vessel that connects the pulmonary artery and the aortic arch in the fetus. Foramen primum is the perforation in the inferior septum primum, which is closed off as it fuses to the endo- cardial cushion. Ductus venosus is the vessel that connects the left umbilical vein to the inferior vena cava. Truncus arteriosus will give rise to the aorta and pulmonary trunk (Fig. 8-5).

A 39-year-old man is admitted to the hospital with odynophagia. A barium swallow reveals an esophageal constriction at the level of the diaphragm. A CT scan and a biopsy further indicate the presence of an esophageal cancer. Which of the following lymph nodes will most likely be affected first? A. Posterior mediastinal and left gastric B. Bronchopulmonary C. Tracheobronchial D. Inferior tracheobronchial E. Superior tracheobronchial

A. Lymph from the lower third of the esophagus drains into the posterior mediastinal and left gastric lymph nodes. The middle third of the esophagus drains into posterior and superior mediastinal lymph nodes. The upper third of the esophagus drains into the deep cervical nodes.

A 2-day-old newborn female is diagnosed with pulmonary artery stenosis, overriding of the aorta, VSD, and hypertrophy of the right ventricle. Which of the following embryologic mechanisms is most likely responsible for the development of this cluster of anomalies? A. Superior malalignment of the subpulmonary infundibulum B. Defect in the aorticopulmonary septum C. Endocardial cushion defect D. Total anomalous pulmonary venous connections E. Atrioventricular canal malformation

A. Superior malalignment of the subpulmonary infundibulum causes stenosis of the pulmonary trunk. This leads to the four symptoms mentioned and is known as tetralogy of Fallot. A defect in formation of the aorticopulmonary septum is characteristic of transposition of the great arteries. An endocardial cushion defect is associated with membranous VSDs

9 A 3-month-old infant is diagnosed with a deletion at the 22q11 chromosome. A routine cardiovascular examination reveals severe congenital cardiac malformation. Which of the following malformations will most likely be associated with 22q11 syndrome? A. Tetralogy of Fallot and truncus arteriosus B. Transposition of the great arteries C. Atrial septal and VSDs D. Coarctation of the aorta E. Aortic atresia

A. Tetralogy of Fallot and truncus arteriosus are associated with DiGeorge syndrome (22q11). Transpo- sition of the great arteries is associated with maternal diabetes. ASDs and VSDs are present in individuals with Down syndrome. Coarctation of the aorta is related to Turner syndrome. Marfan syndrome is present in individuals with aortic atresia.

A 2-day-old newborn female is diagnosed with pulmonary artery stenosis, overriding of the aorta, VSD, and hypertrophy of the right ventricle. Which condition is best characterized by these signs? A. Tetralogy of Fallot B. Atrial septal defect C. Transposition of the great vessels D. Pulmonary atresia E. VSD

A. Tetralogy of Fallot is characterized by four cardiac defects: pulmonary stenosis, VSD, overriding aorta, and these in turn lead to right ventricular hypertrophy. An ASD is characterized by the com- munication between the two atria. In a case of trans- position of the great vessels, the aorta arises from the right ventricle and the pulmonary trunk arises from the left ventricle.

A 35-year-old woman who was brought into the emergency department for a drug overdose requires insertion of a nasogastric tube and administration of activated charcoal. What are the three sites in the esophagus where one should anticipate resistance due to compression on the organ? A. At the aortic arch, the cricopharyngeal constriction, and the diaphragmatic constriction B. The cardiac constriction, the cricoid cartilage constriction, and the thoracic duct C. The pulmonary constriction, cricothyroid constriction, and the azygos vein arch D. The cardiac constriction, the azygos vein arch, and the pulmonary trunk E. The cricopharyngeal constriction, cricothyroid constriction, and thymus gland

A. The esophagus typically has four constrictions. In the thorax the esophagus is compressed by (1) the arch of the aorta, (2) the left principal bron-chus, and (3) the diaphragm. The cricopharyngeal constriction is in the neck.

A 35-year-old man is admitted to the hospital with pain on swallowing. Imaging reveals a dilated left atrium. Which structure is most likely being compressed by the expansion of the left atrium to result in the patient's symptoms? A. Esophagus B. Root of the lung C. Trachea D. Superior vena cava E. Inferior vena cava

A. The patient's chief complaint is pain upon swallowing. With a dilated left atrium, the most prob- able structure being compressed is the esophagus. The esophagus descends into the abdomen immediately posterior to the left atrium below the level of the tracheal carina. The root of the lung is the site of junction at the hilum where the pulmonary arteries, veins, and bronchi enter or leave. The lung root is not so intimately associated with the esophagus and would not be associated with pain during swallowing. The trachea ends and bifurcates above the level of the left atrium and therefore would be unaffected by a dilated left atrium. The inferior vena cava ascends from the abdomen to the right atrium and the superior vena cava is quite anterior in position. Neither of these veins is closely related to the esophagus or the left atrium.

A 25-year-old woman is admitted to hospital with dyspnea. Radiologic examination reveals a tumor invading the lung surface just anterior to the hilum. Which nerve is being compressed by the tumor? A. Phrenic B. Vagus C. Intercostal D. Recurrent laryngeal E. Cardiopulmonary

A. The phrenic nerve passes anterior to the hilum of the lung on both the left and right sides and wraps around the hilar structures inferiorly. The phrenic nerve innervates the diaphragm and if damaged causes dyspnea. The vagus nerve passes along the esophagus as the left and right trunks and is posterior to the hilum on both sides. The intercostal nerves are separated from the tumor by muscle, fat, and fascia. Cardiopulmonary plexus is arranged around the trachea and is located posterior to the hilum of the lung. Wrapping around the arch of the aorta on the left and the subclavian artery on the right, the recurrent laryngeal nerve is far too superior to be affected. None of the other structures cause breathing difficulties.

A 54-year-old man was admitted to hospital for elective esophageal surgery. He had been diagnosed with Barrett's esophagus and opted for surgery that would excise the distal part of the esophagus. Postoperatively, he complained of gradually increasing chest discomfort. Vital signs: P 90 beats/min, BP 160/90, RR 20/min. Auscultation revealed normal heart and breath sounds. A chest radiograph showed hazy opacity of the mediastinum. A CT image of the chest showed a collection in the posterior mediastinum. Which of the following structures is most likely damaged in this patient? A. Thoracic duct B. Esophagus C. Descending aorta D. Azygos vein E. Bronchial lymphatics

A. The thoracic duct originates from the cis- terna chyli in the abdomen and ascends through the aortic hiatus in the diaphragm. It ascends in the pos- terior mediastinum among the thoracic aorta on its left, the azygos vein on its right, the esophagus ante- riorly, and the vertebral bodies posteriorly. At the level of the T4, T5, or T6 vertebra, the thoracic duct crosses to the left, posterior to the esophagus, and ascends into the superior mediastinum.

During a dissection of the posterior mediastinum, a medical student identifies a vessel that lies on the anterior surface of the vertebral bodies between the thoracic aorta on the left and the azygos vein on the right. In a living human being, this vessel would most likely contain which of the following? A. Lymph B. Deoxygenated blood C. Saliva D. Urine E. Oxygenated blood

A. The thoracic duct originates from the cisterna chyli in the abdomen and ascends through the aortic hiatus in the diaphragm. It ascends in the posterior mediastinum among the thoracic aorta on its left, the azygos vein on its right, the esophagus anteriorly, and the vertebral bodies posteriorly. At the level of the T4, T5, or T6 vertebra, the thoracic duct crosses to the left, posterior to the esophagus, and ascends into the superior mediastinum.

49 A 42-year-old man was admitted to the hospital after a head-on vehicular collision in which he received severe blunt trauma to his sternum from the steering wheel. What part of the heart would be most likely to be injured by the impact? A. Right ventricle B. Apex of left ventricle C. Left ventricle D. Right atrium E. Anterior margin of the left atrium

A. These components of the heart are readily viewed in a plain radiograph of the thorax. It is important to understand the spatial arrangement of the heart as it rests in the thorax. The conus region of the right ventricle is located on the most anterior aspect of the heart, thus it is the most anterior portion of the heart within the thorax. The apex of the left ventricle is also located anteriorly, but it is located lateral to the sternum and occupies little area compared with the right ventricle. The left ventricle is positioned on the left lateral side and slightly posterior position in the thorax. The right atrium is located on the right lateral side of the heart. The anterior margin of the left atrium is positioned posteriorly in the thorax.

A 2-year-old child is seen in the pediatric cardiology unit for a congenital heart condition. Which of the following conditions occurs most often? A. Membranous ventricular septal defect (VSD) B. Tetralogy of Fallot C. Muscular VSD D. Ostium secundum defect E. Ostium primum defect

A. VSDs account for 25% of congenital heart defects. The most common of these are defects in the membranous portion of the interventricular septum (membranous VSDs).

40 A 39-year-old man is admitted to the hospital with a complaint of severe retrosternal pain that radiates to the left shoulder. The pain is relieved by leaning forward. Auscultation reveals a pericardial friction rub, leading to a diagnosis of pericarditis. Which of the following nerves is responsible for the radiating pain to the shoulder? A. Intercostobrachial B. Phrenic C. Long thoracic D. Greater thoracic splanchnic E. Cardiopulmonary

B. Pericarditis is an inflammation of the pericar- dium and often causes a pericardial friction rub, with the surface of the pericardium becoming gradually coarser. Because the phrenic nerve is solely respon- sible for innervation of the pericardium, it would transmit the pain fibers radiating from the pericardial friction rub. The phrenic nerve contains sensory nerve fibers from C3 to C5, spinal nerve levels that also supply the skin of the shoulder area; therefore, pain carried by the phrenic nerve may be referred to the shoulder.

A 55-year-old man visits his primary care provider complaining that he can no longer go bike riding without getting breathless and having chest pains. Upon examination, the physician detects a heart murmur. Echocardiography confirms aortic stenosis. What is the embryologic origin of the aorta? A. Bulbus cordis B. Truncus arteriosus C. Sinus venosus D. Ductus arteriosus E. Ductus venosus

B. The arch of the aorta develops from the truncus arteriosus, which is the cranial part of the primitive heart tube. It also gives rise to the pulmo- nary trunk. The bulbus cordis develops into the conus arteriosus of the right ventricle. The sinus venosus has two horns: a right horn that develops into the smooth sinus venarum of the right atrium and a left horn that develops into the coronary sinus. The ductus arteriosus connects the fetal pulmonary trunk to the aorta allowing for blood to bypass the pulmonary circulation. The ductus venosus connects the umbili- cal vein to the inferior vena cava allowing blood to bypass the hepatic circulation in the fetus.

A 35-year-old male bartender is admitted to the hospital due to severe dysphagia. A CT scan (Fig. 2-3) reveals carcinoma of the middle segment of the esophagus. Which of the following structures will most likely be affected if the carcinoma increases greatly in size? A. Inferior vena cava B. Left atrium C. Pulmonary artery D. Left ventricle E. Vertebral body

B. The esophagus lies posterior to the heart. Of the four chambers in the heart, the left atrium lies most posteriorly, just anterior to the esophagus when the heart is in its normal position in the mediastinum. The inferior vena cava runs on the right side within the thoracic cavity and empties its contents into the right atrium. The pulmonary arteries are too anterior to the esophagus to be affected by an esophageal tumor. The left ventricle is too anterior within the mediastinum to be affected by an esophageal tumor. Whereas the esophagus does lie against the vertebral bodies, a growing tumor would affect the esophagus first because it is a smooth muscle structure and therefore the path of least resistance, but this organ can be deviated relatively easily rather than compressed.

7 A 5-year-old boy is admitted to the hospital with severe dyspnea. During physical examination, a loud systolic murmur and a wide, fixed, split S2 sound is noted. What is the most likely diagnosis? A. VSD B. Atrial septal defect C. Tetralogy of Fallot D. Transposition of the great arteries E. Aortic stenosis

B. The murmur at S2 localizes the defect at an atrioventricular valve. An ASD causes a diastolic murmur in the tricuspid valve, whereas a VSD would cause a pansystolic murmur. Transposition of the great arteries and aortic stenosis will cause a murmur at S1, and tetralogy of Fallot does not cause a murmur at S1 or S2.

A 47-year-old man is admitted to the emergency department, due to severe dysphagia. Edema of the left upper limb is apparent upon physical examination. A barium sulfate swallow imaging procedure reveals esophageal dilation, with severe inflammation, due to constriction at the esophageal hiatus from a large lipoma. What is the most likely cause of the severe edema of the left upper limb? A. Thoracic aorta constriction B. Thoracic duct blockage C. Superior vena cava occlusion D. Aortic aneurysm E. Femoral artery disease

B. The thoracic duct is important in lymph drain- age of the entire body with the exception of the upper right quadrant. The thoracic duct ascends between the aorta and azygos vein behind the esophagus. Dilation of the esophagus here in the lower thorax from a large lipoma can compress the thoracic duct, leading to impairment of lymphatic drainage and resultant edema.

10 A 28-year-old woman in her third trimester of pregnancy with a complaint of dizziness for several days is admitted to the hospital. Physical examination reveals that she has diabetes mellitus. Which of the following cardiac malformations is most likely to affect the fetus when the mother has this disease? A. Tetralogy of Fallot B. Transposition of the great arteries C. Atrial septal and VSDs D. Truncus arteriosus E. Coarctation of the aorta

B. Transposition of the great arteries is associated with maternal diabetes. Tetralogy of Fallot and truncus arteriosus are associated with DiGeorge syndrome (22q11). ASDs and VSDs are present in individuals with Down syndrome. Coarctation of the aorta is related to Turner syndrome. Marfan syndrome is present in individuals with aortic atresia.

8 A 3-month-old infant is diagnosed with Down syndrome (trisomy 21). A routine cardiovascular examination reveals that the infant suffers from arrhythmias. What other cardiac conditions are most likely to occur with Down syndrome? A. Tetralogy of Fallot B. Transposition of the great arteries C. Atrial septal and VSDs D. Truncus arteriosus E. Coarctation of the aorta

C. Down syndrome (more properly called "trisomy 21") is associated with cardiovascular abnormalities such as arrhythmias and atrial and VSDs. It is also characterized by mental retardation, brachycephaly, flat nasal bridge, upward slant of the palpebral fissure, protruding tongue, simian crease, and clinodactyly of the fifth digit.

Chest radiographs were taken of a 70-year-old woman who complained of a cough of 3 weeks' duration but was able to ambulate. While reading the films with a medical student, the radiologist asked the student to identify the sternal angle (of Louis). Which of the following radiographic views would best demonstrate this landmark? A. Anteroposterior B. Posteroanterior C. Lateral D. Apical lordotic E. Axial

C. Since the sternal angle (of Louis) is in the transverse plane the best plane to view both the ante- rior and posterior mediastinum, as well as superior mediastinal structures, would be a lateral view. Antero- posterior or posteroanterior would make it difficult separating anterior and posterior structures. Since the sternal angle (of Louis) is in the axial plane an axial cut may miss the level completely.

A 52-year-old man with a long history of alcoholic cirrhosis complained of severe dysphagia and retrosternal "burning" pain. During esophagoscopy, the endoscopist advanced the endoscope until its tip reached the esophageal hiatus of the diaphragm. Which of the following vertebral levels did the tip of the endoscope most likely end? A. T7 B. T8 C. T10 D. T11 E. T12

C. The defects in the diaphragm are at T8 for IVC/phrenic nerve, T10 for esophagus, and T12 for aorta/thoracic duct. T7 and T8 vertebra levels do not have any major diaphragmatic openings.

11 During cardiac catheterization of a 6-year-old child, the radiologist notes that the contrast medium released into the arch of the aorta is visible immediately in the left pulmonary artery. What is the most likely explanation for this finding? A. Atrial septal defect B. Mitral stenosis C. Patent ductus arteriosus D. Patent ductus venosus E. VSD

C. The ductus arteriosus is an embryologic structure that acts as a communication between the pulmonary trunk and the aorta. If it remains patent, the injected contrast medium would flow from the aorta through this communication and into the pul- monary artery. An ASD is a communication between the atria. Mitral stenosis is a narrowing of the AV valve between the left atrium and left ventricle. The ductus venosus transports blood from the left umbili- cal vein to the inferior vena cava, bypassing the liver. A VSD is a communication between the ventricles.

41 A 72-year-old man is admitted to the hospital with complaints of severe chest pain radiating to his left arm. ECG examination provides evidence of significant myocardial infarction of the posterior wall of the left ventricle. Which of the following nerves is responsible for the radiation of pain to the arm during myocardial infarction? A. Phrenic B. Vagus C. Intercostobrachial D. Greater splanchnic E. Suprascapular

C. The intercostobrachial nerve is the lateral cutaneous branch of the second intercostal nerve. It serves a cutaneous function both in the thoracic wall and medial aspect of the arm. The phrenic nerve arises from spinal nerves C3 to C5 and innervates the diaphragm. This nerve has no branches that pass into the arm. The vagus nerve is CN X and supplies auto- nomic function to the gut, up to the left colic flexure, and also provides some autonomic motor and sensory supply to organs in the head, neck, and thorax. The greater thoracic splanchnic nerve originates in the thorax from the sympathetic chain at the levels of T5 to T9 and innervates abdominal structures. The supra- scapular nerve originates from the upper trunk of the brachial plexus and receives fibers primarily from C5 and C6. It innervates the supraspinatus and the infra- spinatus muscles.

A 60-year-old woman with a history of severe rheumatic heart disease is in the clinic for a routine follow-up. Her cardiac symptoms have been relatively stable since valve replacement surgery 2 years ago, but she has developed some new symptoms over the past few months. A chest radiograph reveals sternotomy wires, prosthetic aortic and mitral valves, and a greatly enlarged left atrium. Which symptom could most likely develop as a direct result of her left atrial enlargement? A. Nausea and vomiting B. Pain and tenderness over thoracic vertebral spinous processes C. Difficulty swallowing D. Epigastric pain after eating fatty foods E. Increased coughing

C. The left atrium lies directly anterior to the esophagus and compresses it when enlarged, result- ing in difficulty swallowing. Nausea and vomiting may be present but is a symptom of a wide array of dysfunctions and are not a direct result of the enlarged atrium. Pain and tenderness over the thoracic verte- bral spinous processes will not result because there is no compression of sensory nerves. Epigastric pain resulting from eating fatty foods is an indication of acute cholelithiasis, which is due to gallstones. Increased coughing could only result from irritation of the vagus nerves above the larynx, which is above the level of the left atria.

A 62-year-old male patient expresses concern that his voice has changed over the preceding months. Imaging reveals a growth located within the aortic arch, adjacent to the left pulmonary artery. Which neural structure is most likely being compressed to cause the changes in the patient's voice? A. Left phrenic nerve B. Esophageal plexus C. Left recurrent laryngeal nerve D. Left vagus nerve E. Left sympathetic trunk

C. The left recurrent laryngeal nerve passes beneath the ligamentum arteriosum and then loops superiorly toward the tracheoesophageal groove, medial to the arch of the aorta. It is the only nerve located near the described mass.

A 32-year-old woman is admitted to the emergency department with dyspnea, dysphagia, hoarseness, and severe anxiety. Her medical history reveals that she has lived on a liquid diet for some months and has lost more than 30 lb. Over the past several weeks, she has had bloody sputum during attacks of coughing. Fluoroscopy and a barium swallow reveal a 4-cm mass associated with a bronchus and associated compression of the esophagus. Which of the following nerves is most likely to be affected? A. Right recurrent laryngeal nerve B. Left vagus nerve, posterior to the hilum of the lung C. Left recurrent laryngeal nerve D. Greater splanchnic nerve E. Phrenic nerve

C. The left recurrent laryngeal nerve passes superiorly in the tracheoesophageal groove after looping around the aorta. The compression of this nerve and compression of the esophagus against the trachea would result in the presenting symptoms. The right recurrent laryngeal nerve loops around the right subclavian artery before passing toward the larynx and therefore does not descend into the thorax. The left vagus nerve courses posterior to the hilum of the lung, after it has already given off its left recurrent laryngeal branch at the level of the aortic arch; therefore, compression of this nerve would not result in the presenting symptoms. The greater thoracic splanchnic nerve arises from sympathetic chain ganglia at levels T5 to T9 and therefore would not cause the presenting symptoms. The phrenic nerve innervates the diaphragm; compression of this nerve would not result in the presenting symptoms.

A 2-day-old newborn is diagnosed with transposition of the great arteries. Which structure is primarily responsible for the division of the truncus arteriosus into the great arteries? A. Septum secundum B. Septum primum C. Bulbar septum D. Aorticopulmonary septum E. Endocardial cushions

D. The aorticopulmonary septum functions to divide the truncus arteriosus and bulbus cordis into the aorta and pulmonary trunk. The septum secun- dum forms an incomplete separation between the two atria. The septum primum divides the atrium into right and left halves. The bulbar septum is derived from the bulbus cordis and will give rise to the inter- ventricular septum inferior to the aorticopulmonary septum, eventually fusing with it. The endocardial cushions play a role in the division of the AV canal into right and left halves, by causing the AV cushions to approach each other.

48 A 59-year-old man is admitted to the hospital with severe chest pain. During examination a slight rhythmic pulsation on the chest wall at the left fifth intercostal space is noted in the midclavicular line. What part of the heart is responsible for this pulsation? A. Right atrium B. Left atrium C. Aortic arch D. Apex of the heart E. Mitral valve

D. The apex of the heart is located in the left fifth intercostal space, about 31⁄2 inches to the left of the sternum. When this area of the heart is palpated, any pulsations would be generated by throbbing of the apex of the heart against the thoracic wall. This is also the location for performing auscultation (lis- tening) of the mitral valve, not associated with palpa- tion. The right atrium is located to the right of the sternum. The left atrium is located on the posterior aspect of the heart, thus no direct palpation is real- ized. The aortic arch would be located posterior to the manubrium of the sternum, above the second intercostal space.

A 59-year-old man is admitted to the hospital with severe chest pain. During examination a slight rhythmic pulsation on the chest wall at the left fifth intercostal space is noted in the midclavicular line. What part of the heart is responsible for this pulsation? A. Right atrium B. Left atrium C. Aortic arch D. Apex of the heart E. Mitral valve

D. The apex of the heart is located in the left fifth intercostal space, about 31⁄2 inches to the left of the sternum. When this area of the heart is palpated, any pulsations would be generated by throbbing of the apex of the heart against the thoracic wall. This is also the location for performing auscultation (lis- tening) of the mitral valve, not associated with palpa- tion. The right atrium is located to the right of the sternum. The left atrium is located on the posterior aspect of the heart, thus no direct palpation is real- ized. The aortic arch would be located posterior to the manubrium of the sternum, above the second intercostal space.

36 A 55-year-old man is admitted to the emergency department with a diagnosis of possible myocardial infarction. Which nerves carry pain fibers from the heart to the CNS? A. Vagus B. Greater thoracic splanchnic C. Least thoracic splanchnic D. Cardiopulmonary (thoracic visceral) E. T5 to T9 ventral rami

D. The cardiopulmonary splanchnic (or thoracic visceral) nerves are responsible for carrying the cardiac sympathetic efferent fibers from the sympa- thetic ganglia to the thoracic viscera and afferent fibers for pain from these organs. The vagus nerve is responsible for carrying parasympathetic fibers. The greater and lesser splanchnic nerves carry sympa- thetic preganglionic fibers to the abdomen. T1 to T4 ventral rami receive sensory fibers for pain, carried initially by the cardiopulmonary nerves, en route to their respective final destination.

35 A 23-year-old man is admitted to the emergency department after an automobile collision. Physical examination reveals tachycardia. What is the location of the preganglionic neural cell bodies involved in increasing the heart rate? A. Deep cardiac plexus B. Dorsal motor nucleus of vagus C. Lateral horn T5 to T9 D. Lateral horn T1 to T4 E. Inferior cervical ganglia

D. The lateral horns, or intermediolateral cell columns, contain the cell bodies of preganglionic neurons of the sympathetic system. Spinal cord seg- ments T1 to T4 are often associated with the upper limbs and thoracic organs; the autonomic neurons in spinal cord segments T5 to T9 usually correlate with innervation of organs in the abdominal cavity, specifi- cally organs derived from the foregut.

A 55-year-old man with severe mitral regurgitation is scheduled for minimally invasive mitral valve repair. A percutaneous coronary sinus catheter is placed to deliver retrograde cardioplegia. During the procedure, the catheter stuck in an enlarged the besian valve. Which of the following embryonic structures gives rise to the coronary sinus? A. Primitive ventricle B. Bulbus cordis C. Truncus arteriosus D. Left horn of the sinus venosus-left common cardinal vein E. Primitive atria

D. The left horn of the sinus venosus gives rise to the coronary sinus. The primitive atria and ven- tricles will give rise to the atria and ventricles, respec- tively. Bulbus cordis and truncus arteriosus gives rise to the smooth parts of the right and left ventricles and their corresponding arteries.

A 10-year-old boy is admitted to the hospital with retrosternal discomfort. A CT scan reveals a midline tumor of the thymus gland. Which of the following veins would most likely be compressed by the tumor? A. Right internal jugular B. Left internal jugular C. Right brachiocephalic D. Left brachiocephalic E. Right subclavian

D. The thymus lies in the superior mediastinum and extends upward into the neck, especially in the young. A midline tumor of this gland can compress the left brachiocephalic vein. The subclavian vein is distal or lateral to this location, and the thymus gland would not likely impinge upon it. The internal jugular veins are located superior and lateral to the position of the thymus gland. A midline tumor is more likely to cause compression of the left brachio- cephalic vein, which crosses the midline, than the right brachiocephalic vein, which is not located in the midline.

A 3-week-old prematurely born male neonate with respiratory distress is found to have a systolic murmur crossing the S2 heart sound. The murmur is accompanied by a thrill, is best heard below the left clavicle, and radiates over the chest. On a chest radiograph, the cardiac silhouette is enlarged and pulmonary vascular markings are increased. Echocardiography reveals a congenital defect. A video-assisted thoracoscopic procedure is considered for correction of the underlying defect. Which of the following structures is the most common site of injury? A. Left vagus nerve B. Right vagus nerve C. Left phrenic nerve D. Right phrenic nerve E. Left recurrent laryngeal nerve

E. Due to the long looping course of the left recurrent laryngeal nerve and its location in the superior mediastinum it is more easily damaged during thoracoscopic procedures than the other nerves, which are more posterior and protected from introduction of the thoracoscope. Increased pulmonary vascular markings indicate the presence of a left-to- right shunt. A systolic murmur crossing the S2 heart sound characterizes the continuous (machinery) murmur heard in PDA. Prematurity increases the risk of a PDA. The blood shunt through the patent duct increases with physiological decline in the pulmonary artery pressure towards the end of first month of life.

When inserting a nasogastric tube, which is the most distal site in, or in relation to, the esophagus that might offer resistance to the tube as it passes to the stomach? A. Posterior to the left atrium B. Level of the superior thoracic aperture C. Posterior to the aortic arch D. Posterior to the left main bronchus E. Esophageal hiatus of the diaphragm

E. The esophageal hiatus creates a physiological sphincter during diaphragmatic contraction. As the esophagogastric junction does not have a valve either anatomically or physiologically this would be the last point of resistance. The area posterior to the left atrium may be compressed slightly but would not give resistance to the passing of the tube. At the level of the superior thoracic opening the pharyngoesophageal junction is located and will give resistance; however, this is the first resistance offered by the esophagus. The areas posterior to the aortic arch and left main bronchus are the second and third resistance sites.

A 70-year-old man is admitted to the emergency department for evaluation of his fever, chills, and chest pain. Physical examination reveals a holosystolic murmur that radiates toward the axilla. Blood cultures are ordered and he undergoes TEE. The ultrasound transducer is placed in the midesophagus facing posteriorly. Which of the following structures will be immediately posterior to the transducer? A. Left atrium B. Pulmonary veins C. Right atrium D. Right ventricle E. Aorta

E. The main structure immediately posterior to the esophagus is the descending aorta, and anterior to the esophagus is the left atrium, which is adjacent to the esophagus at this point. The left atrium forms the majority of the posterior surface of the heart and resides adjacent to the esophagus. Enlargement of the left atrium can compress the esophagus and cause dysphagia. The pulmonary veins are too lateral at this point, whereas the right atrium and right ventricle are more anterior.


संबंधित स्टडी सेट्स

Exam 2 Bio Drugs and Society chapter 6

View Set

NUR 200 1.3 Review: Metabolic Alkalosis

View Set

Chapter 46: Management of Patients With Gastric and Duodenal Disorders 4

View Set

The United States 3 (Fill in the Blank)

View Set

[038] Coronary Circulation and Conducting System of Heart

View Set

Chapter 3 Learning Curves 3A, Learning Curves 4D, Learning Curves Chapter 4c, Learning Curves 4b, Chapter 4a Learning Curves, Chapter 3 Learning Curves 3C, Chapter 3 Learning Curves 3B

View Set

International Marketing Chapter 1

View Set